www.vorhilfe.de
- Förderverein -
Der Förderverein.

Gemeinnütziger Verein zur Finanzierung des Projekts Vorhilfe.de.
Hallo Gast!einloggen | registrieren ]
Startseite · Mitglieder · Impressum
Forenbaum
^ Forenbaum
Status VH e.V.
  Status Vereinsforum

Gezeigt werden alle Foren bis zur Tiefe 2

Navigation
 Startseite...
 Suchen
 Impressum
Das Projekt
Server und Internetanbindung werden durch Spenden finanziert.
Organisiert wird das Projekt von unserem Koordinatorenteam.
Hunderte Mitglieder helfen ehrenamtlich in unseren moderierten Foren.
Anbieter der Seite ist der gemeinnützige Verein "Vorhilfe.de e.V.".
Partnerseiten
Weitere Fächer:

Open Source FunktionenplotterFunkyPlot: Kostenloser und quelloffener Funktionenplotter für Linux und andere Betriebssysteme
Forum "Zahlentheorie" - Teileranzahlfunktion Schranke
Teileranzahlfunktion Schranke < Zahlentheorie < Algebra+Zahlentheo. < Hochschule < Mathe < Vorhilfe
Ansicht: [ geschachtelt ] | ^ Forum "Zahlentheorie"  | ^^ Alle Foren  | ^ Forenbaum  | Materialien

Teileranzahlfunktion Schranke: Frage (beantwortet)
Status: (Frage) beantwortet Status 
Datum: 18:01 Mi 07.03.2012
Autor: Dumpfbacke

Aufgabe
6.
a) Man soll zeigen, dass für die Teileranzahlfunktion d(n) die folgende Ungleichung erfüllt ist:

$d(n) < (k [mm] \log [/mm] n [mm] )^{2^{q}}n^{1/q}$, [/mm] für [mm] $q\in \IR [/mm] > 0, [mm] n\ge [/mm] 2 [mm] \in \IN$ [/mm]

Hinweis: Zeigen Sie zuerst dass für ein primes p gilt: falls [mm] $p<2^{q}$ [/mm] dann gilt: [mm] $1+ord_{p}n\le 1+\frac{\log n}{\log p} \le 1+\frac{\log n}{\log 2} \le \frac{2}{\log 2}\log [/mm] n$ und für [mm] $p\ge 2^{q}$ [/mm] gilt: [mm] $1+ord_{p}n \le 2^{ord_{p}n} \le p^{(ord_{p}n)/q}$ [/mm]

b) Man zeige: [mm] $\lim_{n \rightarrow \infty} \frac{\log d(n)}{log(n)} [/mm] = 0$

Hi,

Ich schaffe es bei a) nicht, die Ungleichungen mit den [mm] $ord_{p}n$ [/mm] zu zeigen: [mm] $$1+ord_{p}n\le 1+\frac{\log n}{\log p}$$ [/mm] für [mm] $p<2^{q}$ [/mm]

und [mm] $$1+ord_{p}n \le 2^{ord_{p}n} \le p^{(ord_{p}n)/q}$$ [/mm] für
$p [mm] \ge 2^{q}$. [/mm] Ich sehe auch nicht, wie man dies (also den Hinweis) dann verwenden könnte, um die Ungleichung zu zeigen. Bei b) habe ich angesetzt mit:

[mm] $$\frac{\log d(n)}{n} [/mm] < [mm] \frac{\log d(n)}{\log(n)} [/mm] < [mm] \frac{\log ((\frac{2}{\log 2} \log n)^{2^{q}}n^{1/q})}{\log (n)}$$ [/mm]

Dies führt dann auf [mm] $$\frac{\log ((\frac{2}{\log 2} \log n)^{2^{q}}n^{1/q})}{\log (n)} [/mm] = [mm] \frac{2^{q}\log \frac{2}{\log 2}+ \log \log n - q\log n }{\log n}$$ [/mm]

Und scheint deswegen kein brauchbarer Ansatz zu sein.


Hat jemand eine Idee und mag mir auf die Sprünge helfen?



Mit freundlichen Grüssen:


Dumpfbacke

Ich habe diese Frage in keinem Forum auf anderen Internetseiten gestellt.

        
Bezug
Teileranzahlfunktion Schranke: Antwort
Status: (Antwort) fertig Status 
Datum: 14:45 Fr 09.03.2012
Autor: felixf

Moin!

> 6.
>  a) Man soll zeigen, dass für die Teileranzahlfunktion
> d(n) die folgende Ungleichung erfüllt ist:
>
> [mm]d(n) < (k \log n )^{2^{q}}n^{1/q}[/mm], für [mm]q\in \IR > 0, n\ge 2 \in \IN[/mm]
>  
> Hinweis: Zeigen Sie zuerst dass für ein primes p gilt:
> falls [mm]p<2^{q}[/mm] dann gilt: [mm]1+ord_{p}n\le 1+\frac{\log n}{\log p} \le 1+\frac{\log n}{\log 2} \le \frac{2}{\log 2}\log n[/mm]
> und für [mm]p\ge 2^{q}[/mm] gilt: [mm]1+ord_{p}n \le 2^{ord_{p}n} \le p^{(ord_{p}n)/q}[/mm]
>  
> b) Man zeige: [mm]\lim_{n \rightarrow \infty} \frac{\log d(n)}{log(n)} = 0[/mm]
>  
> Hi,
>
> Ich schaffe es bei a) nicht, die Ungleichungen mit den
> [mm]$ord_{p}n$[/mm] zu zeigen: [mm]1+ord_{p}n\le 1+\frac{\log n}{\log p}[/mm]
> für [mm]$p<2^{q}$[/mm]

Es gilt [mm] $p^{ord_p n} \mid [/mm] n$ und somit [mm] $p^{ord_p n} \le [/mm] n$. Ziehe Logarithmen und dann bist du fast fertig.

> und [mm]1+ord_{p}n \le 2^{ord_{p}n} \le p^{(ord_{p}n)/q}[/mm] für

Fuer die erste Ungleichung zeige allgemein $1 + x [mm] \le 2^x$ [/mm] fuer $x [mm] \ge [/mm] 0$. Das ist echt nicht so schwer (mit vollstaendiger Induktion geht es ganz gut).

Fuer die zweite Ungleichung beachte [mm] $p^{x/q} \ge (2^q)^{x/q} [/mm] = [mm] 2^{q \cdot x/q}$. [/mm]

> [mm]p \ge 2^{q}[/mm]. Ich sehe auch nicht, wie man dies (also den
> Hinweis) dann verwenden könnte, um die Ungleichung zu
> zeigen. Bei b) habe ich angesetzt mit:
>
> [mm]\frac{\log d(n)}{n} < \frac{\log d(n)}{\log(n)} < \frac{\log ((\frac{2}{\log 2} \log n)^{2^{q}}n^{1/q})}{\log (n)}[/mm]

Links kannst du auch einfach $0 [mm] \le \frac{\log d(n)}{\log n}$ [/mm] schreiben. Macht das ganze einfacher.

> Dies führt dann auf [mm]\frac{\log ((\frac{2}{\log 2} \log n)^{2^{q}}n^{1/q})}{\log (n)} = \frac{2^{q}\log \frac{2}{\log 2}+ \log \log n - q\log n }{\log n}[/mm]

Nein, das stimmt so nicht. Schau dir nochmal genau die Rechengesetze fuer Potenzen und Logarithmen an.

LG Felix


Bezug
Ansicht: [ geschachtelt ] | ^ Forum "Zahlentheorie"  | ^^ Alle Foren  | ^ Forenbaum  | Materialien


^ Seitenanfang ^
ev.vorhilfe.de
[ Startseite | Mitglieder | Impressum ]